Answer to Question #196614 in Quantum Mechanics for LUCKY

Question #196614

Suppose, I have normalized the wave function at some point of time. The wave

function evolves with time according to time dependent Schrodinger equation. How do

I know that the wave function remains normalized after some time?

[Hint: Show that 𝑑

𝑑𝑡

∫−∞

|𝜓(𝑥,𝑡)|

2𝑑𝑥 = 0]


1
Expert's answer
2021-05-23T16:36:49-0400

 Schrodinger’s equation says that the dynamical evolution of Ψ is given by

"i ~\\dfrac{\u2202\u03a8}{\u2202t} = \u2212~\\dfrac{\\hbar^2}{2 m}\\dfrac{\u2202^2\u03a8}{\u2202x^2}+ V (x) \u03a8"

for a potential V(x)


Our statistical interpretation is provided by viewing "\u03a8(x, t) \u2217 \u03a8(x, t)" as a density: "\u03c1(x, t)" "= |\u03a8(x, t)|^ 2" so that

"P(a, b) = \\int ^b_a\u03c1(x, t) dx =\\int ^b_a|\u03a8(x, t)|^2dx"


"\\dfrac{d}{dt} \\int^ \u221e_{\u2212\u221e}|\u03a8(x, t)|^2dx =\\int^ \u221e_{\u2212\u221e}\\dfrac{\u2202}{\u2202t} (\u03a8^\u2217(x, t) \u03a8(x, t)) dx"


then we just need the temporal derivative of the norm (squared) of Ψ(x, t):

"\\dfrac{\u2202}{\u2202t} (\u03a8^\u2217(x, t) \u03a8(x, t)) = \\dfrac{\u2202\u03a8^\u2217}{\u2202t} \u03a8 + \u03a8^\u2217 \\dfrac{\u2202\u03a8}{\u2202t}"


Schrodinger’s equation, and its complex conjugate provide precisely the desired connection:

"\\dfrac{\u2202\u03a8}{\u2202t} =\\dfrac{i\\hbar}{ 2 m}\\dfrac{\u2202^2\u03a8}{\u2202x^2}\u2212\\dfrac{i}{\\hbar}~V (x) \u03a8"  


"\\dfrac{\u2202\u03a8^\u2217}{\u2202t} = \u2212\\dfrac{i\\hbar}{ ~2 m}\\dfrac{\u2202^2\u03a8^\u2217}{\u2202x^2}+\\dfrac{i}{\\hbar}~V (x) \u03a8^\u2217"


and we can write the above as:

"\\dfrac{\\partial}{\u2202t} (\u03a8^\u2217(x, t) \u03a8(x, t)) = \\dfrac{i\\hbar}{ ~2 m}\\bigg(\u2212\\dfrac{\u2202^2\u03a8^\u2217}{\u2202x^2}\u03a8 + \u03a8^\u2217\\dfrac{ \u2202^2\u03a8}{\u2202x^2}\\bigg)."


Now, under the integral, we can use integration by parts1 to simplify the above – consider the first term :

"\\int^\u221e_{\u2212\u221e}\\dfrac{\u2202^2\u03a8^\u2217}{\u2202x^2}\u03a8 dx =\\bigg|\\bigg(\\dfrac{\u2202\u03a8^\u2217}{\u2202x} \u03a8\\bigg)\\bigg|^\u221e_{\u2212\u221e}\u2212\\int^ \u221e_{\u2212\u221e}\\dfrac{\u2202\u03a8^\u2217}{\u2202x}\\dfrac{\u2202\u03a8}{\u2202x} dx"


We can get rid of the boundary term by assuming (an additional requirement) that "\u03a8(x, t) \\longrightarrow 0" as "|x| \\longrightarrow\u221e"

"\\dfrac{d}{dt}\\int_{-\\infin}^{\\infin} (\u03a8^\u2217(x, t) \u03a8(x, t)) = \\dfrac{i\\hbar}{ ~2 m}\\int^\\infin_{-\\infin}\\bigg[\\bigg(\u2212\\dfrac{\u2202^2\u03a8^\u2217}{\u2202x^2}\u03a8 + \u03a8^\u2217\\dfrac{ \u2202^2\u03a8}{\u2202x^2}\\bigg)\\bigg]."

"\\dfrac{d}{dt} \\int^ \u221e_{\u2212\u221e}|\u03a8(x, t)|^2dx=\\dfrac{i\\hbar}{ ~2 m}\\int^ \u221e_{\u2212\u221e}\\bigg[\\dfrac{\u2202\u03a8^\u2217}{\u2202x}\\dfrac{\u2202\u03a8}{\u2202x}-\\dfrac{\u2202\u03a8^*}{\u2202x}\\dfrac{\u2202\u03a8}{\u2202x}\\bigg] dx"

"\\dfrac{d}{dt} \\int^ \u221e_{\u2212\u221e}|\u03a8(x, t)|^2dx=0"




Need a fast expert's response?

Submit order

and get a quick answer at the best price

for any assignment or question with DETAILED EXPLANATIONS!

Comments

No comments. Be the first!

Leave a comment

LATEST TUTORIALS
New on Blog
APPROVED BY CLIENTS